Thứ Sáu, 25 tháng 3, 2016

Mở rộng Canada 1979

Problem : Cho $x_0<x_1<...<x_n$ là các số tự nhiên. Chứng tỏ :
$A=\frac{1}{[x_0,x_1]}+\frac{1}{[x_1,x_2]}+...\frac{1}{[x_n,x_{n-1}]} \le \frac{2^n-1}{2^n}$
                                                                Lời giải :
Ta sẽ chứng minh nó bằng quy nạp
+Với $n=1$ thì bạn đọc tự chứng minh bất đẳng trên đúng
+Giả  sử bất đẳng thức trên đúng trên với $n \ge 1$
Ta sẽ chứng minh trên cũng đúng với $n+1$
Ta xét $2$ trường hợp :
Trường hợp 1 :$x_{n+1} \ge 2^{n+1} \Rightarrow [x_n,x_{n+1}] \ge x_{n+1} \ge 2^{n+1}$
Theo giả thiết quy nạp thì :
$A+\frac{1}{[x_n,x_{n+1}]} \le \frac{2^n-1}{2^n}+\frac{1}{2^{n+1}}=\frac{2^{n+1}-1}{2^{n+1}}$
Trường hợp 2 : $x_{n+1}<2^{n+1}$
Ta có đẳng thức sau $[a,b].(a,b)=ab$ . Do $(a,b)|b$ nên suy ra :
$\frac{1}{[a,b]}=\frac{(a,b)}{ab} \le \frac{1}{a}-\frac{1}{b}$
Nên : $A+\frac{1}{[x_n,x_{n+1}]} \le \frac{1}{x_0}-\frac{1}{x_1}+\frac{1}{x_1}-\frac{1}{x_2}+...+\frac{1}{x_n}-\frac{1}{x_{n+1}}$
$\Rightarrow \frac{1}{x_0}-\frac{1}{x_{n+1}}<1-\frac{1}{2^{n+1}}=\frac{2^{n+1}-1}{2^{n+1}}$
Bất đẳng thức được chứng minh.

Thứ Sáu, 18 tháng 3, 2016

$x!+y!=3^n.n!$

Problem : Tìm bộ ba nghiệm tự nhiên của phương trình : $x!+y!=3^n.n!$
                                                                Lời giải : 
Ta gọi phương trình trên là phương trình (1). Dễ thấy $n \ge 1$ . Vì vai trò $x,y$ bình đẳng nên ta có thể giả sử $y \ge x$
Trường hợp 1 : $x \le n$
Ta có (1) $\Leftrightarrow 1+\frac{y!}{x!}=\frac{3^n.n!}{x!}$ (2)
Suy ra $1+\frac{y!}{x!} \equiv 0 \pmod{3}$ . Để ý tích $3$ số nguyên liên tiếp chia hết cho $3$ và $n \ge 1$. Suy ra $x<y \le x+2$
-Nếu $y=x+2$ thì từ (2) ta có $1+(x+1)(x+2)=\frac{3^n.n!}{x!}$ (3) . Để ý tích $2$ số nguyên liên tiếp chia hết cho $2$ . Từ (3) suy ra $n \le x+1$ nên với $n=x$ thì từ (3) ta được  $(x+1)(x+2)+1=3^x$ hay $x^2+3x+3=3^x$ (4)
Vì $x \ge 1$ nên theo (4) thì $x \equiv 0 \pmod{3} \Rightarrow x \ge 3$ .Từ (4) ta có
$-3=x^2+3x-3^x \equiv 0 \pmod{9}$ (vô lí). Suy ra $n \ne x$
Với $n=x+1$ từ (3) ta có $1+(x+1)(x+2)=3^n(x+1)$ . Từ đó suy ra $x=0$ dẫn đến $y=2$ và $n=1$
-Nếu $y=x+1$ thì từ (2) ta có : $x+2=3^n.\frac{n!}{x!}$ (5) . Cộng với việc để ý $n \ge 1$ thì theo (5) ta thấy ngay $x \ge 1$ . Do đó $x+2 \equiv 1 \pmod{x+1}$ . Từ đó theo (5) dẫn đến việc $n=x$
Ta đưa về phương trình theo (5) $\Leftrightarrow x+2=3^x$ . Đến đây ta dễ dàng thu được $(x,y,n)=(1,2,1)$
Trường hợp 2 : $x>n$
Khi đó (1) $\Leftrightarrow \frac{x!}{n!}+\frac{y!}{n!}=3^n$ (7) . Để ý rằng $n+1,n+2$ không thể đồng  thời là lũy thừa của $3$ nên từ (7) suy ra $x=n+1$
Phương trình được viết lại thành : $n+1+\frac{y!}{n!}=3^n$ (8). Vì $y \ge x \Rightarrow y \ge n+1$
Đặt $A=\frac{y!}{(x+1)!}$
Khi đó có thể viết (8) lại thành $(n+1)(1+A)=3^n$ (9)
Rõ ràng nếu $y \ge n+4$ thì $A \equiv 0 \pmod{4}$ vì thế $A+1$ không thể là một lũy thừa của $3$ nên từ (9) suy ra $y \le n+3$
Nếu $y=n+3$ thì $A=(n+2)(n+3)$ nên từ (9) ta có $(n+1)[1+(n+2)(n+3)]=3^n$ hay $(n+2)^3-1=3^n$ (10). Ta đặt $n+2=3k+1$ với $k \ge 2$ khi đó (10) được viết lại thành $9k(3k^2+3k+1)=3^{3k-1}$ vô lí vì $3k^2+3k+1$ không là lũy thừa của $3$
Nếu $y=n+2$ thì  $A=n+2$ do đó từ (9) ta có $(n+1)(n+3)=3^n$ (dễ thấy phương trình vô nghiệm vì $gcd(n+1,n+3)=1$)
Nếu $y=n+1$ thì ta có $A=1$ từ (9) ta có $2(n+1)=3^n$ vô lí vì một bên lẻ một bên chẵn
Kết luận : Vậy $(x,y,n)=(0,2,1);(2,0,1);(1,2,1);(2,1,1)$

Thứ Năm, 17 tháng 3, 2016

APMO 2011

Problem : Cho các số nguyên $a,b,c$. Chứng minh rằng không thể đồng thời xảy ra đồng thời ba số $a^2+b+c,b^2+a+c,c^2+a+b$ đều là những số chính phương.
                                                                Lời giải :
Vì $a,b,c$ có tính đối xứng nhau nên ta có thể giả sử $a \ge b \ge c$
Ta có $a^2+b+c>a^2$ nên suy ra $a^2+b+c \ge (a+1)^2$ (do $a^2+b+c$ là số chính phương)
Suy ra $b+c \ge 2a+1$. Mà ta có $2a \ge b+c$ suy ra $2a \ge 2a+1$ (vô lí)
Hoàn tất chứng minh.

Thứ Tư, 16 tháng 3, 2016

Balkan MO 2005

Problem : Tìm tất cả các số nguyên tố $p$ sao cho $p^2-p+1$ là lập phương của một số tự nhiên.
Đặt $p^2-p+1=a^3$ với $a \in \mathbb{N^*}$
Khi đó $p(p-1)=(a-1)(a^2+a+1)$ (1)
Xét  trường hợp nếu $a=pk+1$ với $k \in \mathbb{N}$
Từ (1) ta có $p^2-p+1=(pk)^3+(pk)^2+3(pk)+1$ với $k \ge 2$ thì $VP>VT$ do đó $k=0$ hoặc $k=1$
Với $k=0$ thì suy ra $p(p-1)=0$ (vô lí)
$k=1$ thì $p^2-2p+4=0$ . Phương trình này vô nghiệm
Trường hợp $p|(a^2+a+1)$ (2) ta có theo (1) thì $a-1|p(p-1)$  suy ra $a-1|(p-1)$ do $gcd(p,a-1)=1$
Đặt $p=(a-1).z+1$ với $z$ là số tự nhiên. Theo (2) thì $\frac{a^2+a+1}{az-z+1} \in \mathbb{Z}$
Hay $(a^2+a+1)z \vdots ((a-1)z+1)$
Hay $(az-z+1)|(3z-2-a)$ (3)
Ta chia thành các trường hợp sau :
Trường hợp 1 : $3z-2-a \ge az-z+1 \Leftrightarrow z(4-a) \ge a+3$
Dễ thấy $2 \le a \le 3$ . Nếu $a=2$ thì $p(p-1)=7$ phương trình này vô nghiệm nguyên tố
$a=3$ thì $p(p-1)=26$ phương trình này cũng vô nghiệm nguyên tố
Trường hợp 2 : $2+a-3z \ge az-z+1 \Leftrightarrow z(2+a) \le a+1$ vô lí do $2+a>a+1$
Trường hợp 3 : $3z-2-a=0$ nên $a=3z-2$ khi đó $p=3z(z-1)+1$ và $a^2-a+1=9z^2-9z+3$
Suy ra $a^2+a+1=3p$
Từ (1) ta có $p-1=3(a-1) \Leftrightarrow z=1$ hoặc $zz=3$
Nếu $z=1$ thì dễ thấy trường hợp này vô lí
$z=3$ thì $p=3a-2$ khi đó (1) $\Leftrightarrow 9a^2-15a+7=a^3$
Giải phương trình này cho ta $a=1$ hoặc $a=7$
Thử lại cho ta $a=7$ thỏa mãn vì khi đó thì $p=19$
Vậy số nguyên tố $p$ thỏa mãn bài toán là $p=19$

Thứ Ba, 15 tháng 3, 2016

Bài toán đầu tiên

Problem :Cho $a,b,c,d \in \mathbb{Z^+}$ thỏa mãn $(ac+bd) \vdots (a^2+b^2)$ .Chứng minh rằng $gcd(a^2+b^2,c^2+d^2)>1$ 
                                                                    Lời giải :  
Chú ý rằng $(ac+bd)^2+(ad-bc)^2=(a^2+b^2)(c^2+d^2)$ (1)
Giả sử $gcd(a^2+b^2,c^2+d^2)=1$ (2)
Trường hợp 1 : $a^2+b^2$ không phải là số chính phương. Khi đó tồn tại ước nguyên tố $p$ có số mũ là $2n+1$ với $n$ là số tự nhiên
Từ (1) suy ra $(ad-bc)^2 \vdots (a^2+b^2) \vdots p^{2n+1} \Rightarrow (ad-cb)^2 \vdots p^{2n+2}$
Ta chú ý rằng : $(ac+bd)^2 \vdots p^{4n+2} \vdots p^{2n+2}$
$VT(1) \vdots p^{2n+2} \Rightarrow (a^2+b^2)(c^2+d^2) \vdots p^{2n+2}$
Kết hợp với (2) suy ra $gcd(p,c^2+d^2)=1 \Rightarrow gcd(p^{2n+2},c^2+d^2)=1$
Khi đó $p^{2n+2}|(a^2+b^2)$ (vô lí)
Trường hợp 2 : $a^2+b^2$ là một số chính phương . Đặt $a^2+b^2=t^2$ trong đó $t \in \mathbb{N^*}$
Ta có $(a^2+b^2)|(ac+bd) \Rightarrow ac+bd=t^2.x$ ($x \in \mathbb{N^*}$)
Từ (1) suy ra $t^2|(ad-bc)^2 \Rightarrow t|(ad-bc) \Rightarrow ad-bc=ty$ trong đó $y$ là số nguyên dương
Từ đó ta có $d(ac+bd)-c(ad-bc)=dxt^2-tyc$
Hay $b(c^2+d^2)=t(dxt-cy)$ suy ra $t|b(c^2+d^2)$
Mà vì $gcd(c^2+d^2,t^2)=1$ (theo điều giả sử) nên $gcd(c^2+d^2,t)=1$
Suy ra $b \vdots t$ suy ra $b \ge t \Leftrightarrow b \ge \sqrt{a^2+b^2}$ (vô lí)
Vậy điều giả sử của ta là sai. Ta có điều phải chứng minh.